[HN Gopher] The seventh most popular easily understood unsolved ...
       ___________________________________________________________________
        
       The seventh most popular easily understood unsolved problem on
       MathOverflow
        
       Author : ColinWright
       Score  : 61 points
       Date   : 2024-02-16 19:01 UTC (3 hours ago)
        
 (HTM) web link (mathstodon.xyz)
 (TXT) w3m dump (mathstodon.xyz)
        
       | nomemory wrote:
       | It still boggling we haven't proved the sum of pi + e is
       | irrational. Who cares anyway?
        
         | xanderlewis wrote:
         | > Who cares anyway?
         | 
         | I can't tell if you're just joking. Proofs of facts that seem
         | obvious or irrelevant are very important not for the yes-no
         | question they answer but for the development of the deep
         | mathematical theory they necessitate.
        
           | nomemory wrote:
           | It was a joke, I know there are lots of people who care about
           | that.
        
         | SOTGO wrote:
         | It's not that mind-boggling. In general we need to use specific
         | properties of numbers to prove whether they are irrational, and
         | pi+e has very few useful properties to work from. Even proving
         | pi is irrational is not trivial and it is one of the
         | transcendental numbers we know the most about.
        
           | xanderlewis wrote:
           | > we need to use specific properties of numbers to prove
           | whether they are irrational
           | 
           | Indeed, one could say that _all_ we know about numbers are
           | their properties. A number is just an existence assertion
           | about an object fulfilling some property (possibly uniquely).
        
       | nerdponx wrote:
       | > fairly simple
       | 
       |  _That 's_ your definition of a fairly simple identity?!
        
         | nomemory wrote:
         | In his defense lots of unsolved questions are inscrutable for
         | the majority of people. So this one looks decent.
        
         | Almondsetat wrote:
         | it fits in a line. it's simple
        
         | ajkjk wrote:
         | Well it's something an undergraduate can understand and in
         | principle compute/play with. That's pretty simple,
         | comparatively.
        
       | Buttons840 wrote:
       | One of the comments was "the Collatz conjecture feels like we're
       | missing a branch of mathematics", or something to that effect. I
       | followed that rabbit hole a bit and found the Plya conjecture[0],
       | which was disproven when a counter example was found at
       | approximately 10^361. Here I was naively thinking that if no
       | counter examples were found in the first, say, 10^10 numbers, no
       | counter examples should exist. If only it were so easy.
       | 
       | [0]: https://en.wikipedia.org/wiki/P%C3%B3lya_conjecture
        
         | messe wrote:
         | That's just the first counterexample that was proven to exist.
         | The smallest counter example is less than 10^10 (in fact
         | smaller than 10^9).
         | 
         | > The Polya conjecture was disproved by C. Brian Haselgrove in
         | 1958. He showed that the conjecture has a counterexample, which
         | he estimated to be around 1.845 x 10^361.[3]
         | 
         | > An explicit counterexample, of n = 906,180,359 was given by
         | R. Sherman Lehman in 1960;[4] the smallest counterexample is n
         | = 906,150,257, found by Minoru Tanaka in 1980.[5]
         | 
         | EDIT: rephrased because despite having a maths degree I can't
         | count.
        
           | klysm wrote:
           | I believe inability to do arithmetic correctly is more common
           | in folks with maths and other STEM degrees
        
             | messe wrote:
             | I do like to joke that I studied maths because I don't like
             | numbers.
        
               | at_a_remove wrote:
               | [delayed]
        
             | DavidSJ wrote:
             | It's one of the prerequisites in fact.
        
           | cubefox wrote:
           | > He showed that the conjecture has a counterexample, which
           | he estimated to be around 1.845 x 10361
           | 
           | 1.845 x 10361 = 19116.045
        
             | messe wrote:
             | Thanks, it should be 10^361. I've fixed it.
        
         | senderista wrote:
         | See also https://mathoverflow.net/questions/95865/examples-of-
         | conject...
        
         | Shawnecy wrote:
         | That reminds me of a parody song from 3Blue1Brown[0] about how
         | certain numerical patterns don't quite hold and can fool you if
         | you don't look far enough out.
         | 
         | [0]: https://www.youtube.com/watch?v=NOCsdhzo6Jg
        
         | SilasX wrote:
         | Okay but the Collatz conjecture is a little different in that
         | there can't just be a one-off counterexample: it's a statement
         | about a sequence. The counterexample would have to be either a
         | cycle (that excludes 4/2/1), or a sequence of numbers that keep
         | spiraling up indefinitely. And they've proven that any cycle
         | would have to be very long[1]. Either way, it would mean
         | trivially unlocking a sequence of numbers that happens to
         | satisfy a very specific criterion.
         | 
         | [1]
         | https://en.wikipedia.org/wiki/Collatz_conjecture?useskin=vec...
        
           | timeagain wrote:
           | Right. So there might be some set of numbers way out there
           | that are like the "sporadic simple group(s)" of the collatz
           | conjecture. Following the same rules as everyone else, you
           | end up with something otherwise unclassifiable.
        
         | eutectic wrote:
         | John Conway showed that if you generalise the coefficients of
         | the Collatz conjecture then some instances are undecidable. So
         | maybe there is no proof.
        
           | cubefox wrote:
           | If it is "undecidable" this means there is no counterexample
           | to the Collatz conjecture, since any counterexample would
           | disprove it. But the Collatz conjecture does exactly state
           | that there are no counterexamples. Which means: If it is
           | undecidable, it is true.
           | 
           | Which seems a bit paradoxical. If you can prove that the
           | Collatz conjecture is undecidable, you would also prove that
           | it has no counterexamples, and thus that it is true. Which
           | would make it decidable -- contradiction. So this seems to
           | prove that if the Collatz conjecture is undecidable, this
           | fact is itself also undecidable.
        
             | dooglius wrote:
             | You missed the key part
             | 
             | > if you generalise the coefficients
             | 
             | The result appears to be
             | https://gwern.net/doc/cs/computable/1972-conway.pdf if you
             | want to read in detail
        
             | nickdrozd wrote:
             | > If it is undecidable, it is true.
             | 
             | That is the case for something like Goldbach's Conjecture,
             | which says that every even number > 2 is the sum of two
             | primes. If it's false, then there is a counterexample, and
             | it is easy to prove whether or not a given number is a
             | counterexample (just loop over all pairs of smaller
             | primes).
             | 
             | But that is not the case for the Collatz Conjecture. A
             | Collatz counterexample could be a number whose orbit loops
             | back around. That would be a provable counterexample.
             | Another kind of Collatz counterexample would be a number
             | whose orbit never terminates or repeats, it just keeps
             | going forever. If such an infinite sequence existed, it
             | might not be possible to prove that it's infinite. And if
             | it isn't provable, then the conjecture would both
             | undecidable and false.
        
           | thesz wrote:
           | Do you have a link?
        
         | permo-w wrote:
         | >Here I was naively thinking that if no counter examples were
         | found in the first, say, 10^10 numbers, no counter examples
         | should exist
         | 
         | why on earth would you ever think that
        
           | zimpenfish wrote:
           | > naively
           | 
           | I think that answers your question?
        
         | pulisse wrote:
         | There's a great Mathoverflow thread collecting examples of this
         | sort of phenomenon (called "eventual counterexamples" in the
         | thread), together with some interesting analysis (see Joel
         | Hamkins' answer):
         | https://mathoverflow.net/questions/15444/examples-of-eventua...
        
       | coolThingsFirst wrote:
       | Im in the middle of something but that doesn't look that hard
        
       | paulpauper wrote:
       | Gourevitch's conjecture is based on a transformed Ramanujan pi
       | formula via Dougall's identity .
       | 
       | more info
       | 
       | https://www.ams.org/journals/mcom/2014-83-285/S0025-5718-201...
        
       | screenoridesaga wrote:
       | I love how these raw mathematicians consider something proved
       | when they can understand, meanwhile the computer can prove it
       | easily just by counting a finite number of bits. What exactly
       | would be considered proof in this case? Any explanation only
       | mathematicians can understand?
        
         | earlymodernlvr wrote:
         | The computer can only check a finite number of cases
         | computationally, a proof can prove the results for all N.
        
         | red_trumpet wrote:
         | Well, how does your proposed computer proof look like? A
         | computer can easily calculate both sides of the equation up to
         | say, float precision. But that's not a proof; it only tells you
         | that both numbers are near each other!
        
         | feoren wrote:
         | > the computer can prove it easily just by counting a finite
         | number of bits
         | 
         | Did you miss the infinite sum there? How would you prove an
         | infinite sum equals a transcendental number by counting finite
         | bits? You'd have to count _infinite_ bits.
        
           | hyperhello wrote:
           | Wouldn't you be able to see the difference converging on zero
           | at least? Unless it oscillates all over but seemed to
           | average. I don't know if the squeeze theorem applies.
        
             | tsimionescu wrote:
             | There are many series that converge to an extraordinarily
             | small but non 0 number.
        
         | damiankennedy wrote:
         | The first thing you need to go with a series like this is prove
         | that it converges. Then you can take a series that is already
         | known to converge to pi. The choice of series will mean the
         | difference between the proof being very hard and practically
         | impossible. Then change that series to give 32/pi^3 instead of
         | pi. Then deduct a mapping between groups of n in one series and
         | groups of n in the other series.
        
       | datameta wrote:
       | This may be a question that misses the point - respectfully, what
       | are some practical applications in physics or engineering for
       | such proofs and/or the search for a conjecture counterexample?
        
         | macrolocal wrote:
         | Sorry for the cliche, but "the most exciting phrase to hear in
         | science, the one that heralds the most discoveries, is not
         | "Eureka!" (I found it!) but 'That's funny...'" --Asimov
        
         | Sharlin wrote:
         | None that we know of. We still do it because it's worth doing.
        
       | tzs wrote:
       | Here's a fairly easily understood problem, which if you could
       | solve it would make you famous in the mathematical world and win
       | you a million dollar prize:                 For a positive
       | integer n:              Let H(n) = 1 + 1/2 + 1/3 + ... + 1/n
       | Let D(n) = the sum of the positive integers that divide n. E.g.,
       | D(12) = 1 + 2 + 3 + 4 + 6 + 12 = 28.            Prove or disprove
       | that for any positive integer n > 1:              D(n) < H(n) +
       | exp(H(n)) log(H(n))
       | 
       | That easy to understand problem turns out to be equivalent to the
       | Riemann hypothesis [1], which is one of the most famous and
       | important unsolved problems in number theory.
       | 
       | [1] https://arxiv.org/abs/math/0008177
        
       ___________________________________________________________________
       (page generated 2024-02-16 23:00 UTC)